You are on page 1of 33

Petitioner: AKLAN NATIONAL COLLEGES OF FISHERIES (ANCF)

Respondent: HEIRS OF MAXIMA LACHICA SIN


G.R. NO. 157485

Facts:
Respondent heirs instituted a complaint against the Aklan National College of fisheries (ANCF) in Kalibo, aklan), for
recovery of possession, quieting of title, and declaration of ownership with damages. Respondent heirs claim that a
41,231-square meter-portion of the property they inherited had been usurped by ANCF, creating a cloud of doubt with
respect to their ownership over the parcel of land they wish to remove from the ANCF reservation.

The respondent heirs presented evidence that they inherited a bigger parcel of land from their mother, Maxima Sin, who
died in the year 1945 in New Washington, Capiz (now Aklan). Maxima Sin acquired said bigger parcel of land by virtue of a
Deed of Sale, and then developed the same by planting coconut trees, banana plants, mango trees and nipa palms and
usufructing the produce of said land until her death in 1945.

Respondent heirs of Maxima Sin asserted that they were previously in possession of the disputed land in the concept of an
owner. To prove possession, respondents presented several tax declarations, the earliest of which was in the year 1945.

The ANCF Superintendent countered that the parcel of land being claimed by respondents was the subject of Proclamation
No. 2074 of then President Ferdinand E. Marcos allocating 24.0551 hectares of land within the area, civil reservation for
educational purposes of ANCF and that the subject parcel of land is timberland and therefore not susceptible of private
ownership.

MTC
On June 19, 2000, the MCTC rendered its Decision in favor of respondents Heirs. The MCTC thus ruled that the claim of
respondent heirs over the disputed land by virtue of their and their predecessors’ open, continuous, exclusive and
notorious possession amounts to an imperfect title, which should be respected and protected.

RTC and CA
The RTC and CA affirmed the MCTC’s decision.

Issue:
Whether or not the heirs of Maxima Lachica Sin acquired private rights by virtue of their possession to the disputed
property so as to entitle them for the judicial confirmation of imperfect title.

Ruling:
No, the rights of the heirs by virtue of their possession does not entitle them for the judicial confirmation of title, because
one of the requirements under Property Registration Decree is that the land must be classified as alienable and disposable
land of the public domain. They cannot prove that such land was classified as such during the period of possession.

RATIO:
The requirements for judicial confirmation of imperfect title are found in Section 48(b) of the Public Land Act, as amended
by Presidential Decree No. 1073, as follows:
Sec. 48. The following described citizens of the Philippines, occupying lands of the public domain or claiming
to own any such lands or an interest therein, but whose titles have not been perfected or completed, may
apply to the Court of First Instance of the province where the land is located for confirmation of their claims
and the issuance of a certificate of title therefor, under the Land Registration Act, to wit:
xxxx
(b) Those who by themselves or through their predecessors in interest have been in the open, continuous,
exclusive, and notorious possession and occupation of alienable and disposable lands of the public domain,
under a bona fide claim of acquisition or ownership, since June 12, 1945, or earlier, immediately preceding
the filing of the application for confirmation of title except when prevented by war or force majeure. These
shall be conclusively presumed to have performed all the conditions essential to a Government grant and
shall be entitled to a certificate of title under the provisions of this chapter.

An equivalent provision is found in Section 14(1) of the Property Registration Decree, which provides:
SECTION 14. Who may apply.— The following persons may file in the proper Court of First Instance an
application for registration of title to land, whether personally or through their duly authorized
representatives:
(1) those who by themselves or through their predecessors-in- interest have been in open, continuous,
exclusive and notorious possession and occupation of alienable and disposable lands of the public domain
under a bona fide claim of ownership since June 12, 1945, or earlier.
This Court has thus held that there are two requisites for judicial confirmation of imperfect or incomplete
title under CA No. 141, namely:
(1) open, continuous, exclusive, and notorious possession and occupation of the subject land by
himself or through his predecessors-in-interest under a bona fide claim of ownership since time
immemorial or from June 12, 1945; and
(1) the classification of the land as alienable and disposable land of the public domain.

With respect to the second requisite, ANCF objected that property was alienable and disposable before 1960 (date of
proclamation that land is reserved for ANCF), because under the Regalian Doctrine, all lands of the public domain belong
to the State and that lands not appearing to be clearly within private ownership are presumed to belong to the State.

Valiao v. Republic:
Under the Regalian doctrine, which is embodied in our Constitution, all lands of the public domain belong to the State,
which is the source of any asserted right to any ownership of land. All lands not appearing to be clearly within private
ownership are presumed to belong to the State. Occupation thereof in the concept of owner no matter how long cannot
ripen into ownership and be registered as a title.To overcome this presumption, incontrovertible evidence must be
established that the land subject of the application (or claim) is alienable or disposable.

To prove that the land subject of an application for registration is alienable, the applicant must establish the existence of a
positive act of the government, such as a presidential proclamation or an executive order; an administrative action;
investigation reports of Bureau of Lands investigators; and a legislative act or a statute. The applicant may also secure a
certification from the government that the land claimed to have been possessed for the required number of years is
alienable and disposable.

Secretary of the DENR v. Yap:


A positive act declaring land as alienable and disposable is required. In keeping with the presumption of State ownership,
the Court has time and again emphasized that there must be a positive act of the government, such as an official
proclamation, declassifying inalienable public land into disposable land for agricultural or other purposes.In fact, Section 8
of CA No. 141 limits alienable or disposable lands only to those lands which have been "officially delimited and classified."

Heirs of the Late Spouses Pedro S. Palanca and Soterranea Rafols v. Republic:
In the absence of the classification as mineral or timber land, the land remains unclassified land until released and
rendered open to disposition.

Heirs of Mario Malabanan v. Republic:


The members of this Court were in disagreement as to whether lands declared alienable or disposable after June 12, 1945
may be subject to judicial confirmation of imperfect title. There was, however, no disagreement that there must be a
declaration to that effect.

In the case at bar, it is therefore the respondents which have the burden to identify a positive act of the government, such
as an official proclamation, declassifying inalienable public land into disposable land for agricultural or other purposes.
Since respondents failed to do so, the alleged possession by them and by their predecessors-in-interest is inconsequential
and could never ripen into ownership. Accordingly, respondents cannot be considered to have private rights within the
purview of Proclamation No. 2074 as to prevent the application of said proclamation to the subject property. We are thus
constrained to reverse the rulings of the courts a quo and grant the prayer of petitioner Republic to dismiss Civil Case for
lack of merit.

REVERSED and SET ASIDE.


NOTES:

It must be noted that respondents have not filed an application for judicial confirmation of imperfect title under the Public
Land Act or the Property Registration Decree. Nevertheless, the courts a quo apparently treated respondents’ complaint
for recovery of possession, quieting of title and declaration of ownership as such an application and proceeded to
determine if respondents complied with the requirements therefor.

Description of land owned by Sin: A parcel of cocal, nipal and swampy land, located at Barangay Tambac, New
Washington, Aklan, 58,606 square meters, with an assessed value of Php1,320.00.2

Action filed: Recovery of possession, quieting of title, and declaration of ownership with damages, filed by heirs of Maxima
Sin against Lucio Arquisola, Superintendent of ANCF for.

Amendment in the complaint: Subsequently, the complaint was amended to include ANCF as a party defendant and Lucio
Arquisola, who retired from the service during the pendency of the case, was substituted by Ricardo Andres, then the
designated Officer-in-Charge of ANCF.

The RTC remanded the case to the MCTC of New Washington and Batan, Aklan, in view of the enactment of Republic Act
No. 7659 which expanded the jurisdiction of first-level courts. The case was docketed as Civil Case No. 1181 (4390).

MCTC decision:
the segregation of the same from the Civil Reservation of the Aklan National College of Fisheries, actual damages for the
unearned yearly income from nipa plants uprooted by the defendants in the amount of Php3,500.00 yearly beginning the
year 1988 until plaintiffs are fully restored to the possession of the land in question.

Reason of MCTC:
 sketch made by the Court Commissioner in his report shows that the disputed property is an alienable and disposable
land of the public domain. Furthermore, the land covered by Civil Reservation under Proclamation No. 2074 was
classified as timberland only on December 22, 1960. The MCTC observed that the phrase "Block II Alien or Disp. LC
2415" was printed on the Map of the Civil Reservation for ANCF established under Proclamation No. 2074, indicating
that the disputed land is an alienable and disposable land of the public domain.
 Republic v. Court of Appeals:  where it was pronounced that “Lands covered by reservation are not subject to entry,
and no lawful settlement on them can be acquired. The claims of persons who have settled on, occupied, and
improved a parcel of public land which is later included in a reservation are considered worthy of protection and are
usually respected, but where the President, as authorized by law, issues a proclamation reserving certain lands, and
warning all persons to depart therefrom, this terminates any rights previously acquired in such lands by a person
who has settled thereon in order to obtain a preferential right of purchase. And patents for lands which have been
previously granted, reserved from sale, or appropriated are void.”

Noting that there was no warning in Proclamation No. 2074 requiring all persons to depart from the reservation, the MCTC
concluded that the reservation was subject to private rights if there are any.
The MCTC thus ruled that the claim of respondent heirs over the disputed land by virtue of their predecessors’ open,
continuous, exclusive and notorious possession amounts to an imperfect title, which should be respected and protected.

RTC
 modified absolving Appellant Ricardo Andres from the payment of damages and attorney’s fees.
 All other details of the appealed decision are affirmed in toto.

Reason of RTC:
 Proclamation No. 2074 recognizes vested rights acquired by private individuals prior to its issuance on March 31,
1981.

CA
 dismissing the petition for lack of merit.

Reason of CA
 ANCF had not shown by competent evidence that the subject land was likewise declared a timberland before its
formal classification as such in 1960. Considering that lands adjoining to that of the private respondents, which are
also within the reservation area, have been issued original certificates of title, the same affirms the conclusion that
the area of the subject land was agricultural, and therefore disposable, before its declaration as a timberland in 1960.
 Maxima Lachica Sin acquired, through purchase and sale, from its previous owners spouses Sotera Melocoton and
Victor Garcia in 1932, or 28 years before the said landholding was declared a timberland in 1960. (subject property
had been privately possessed for more than 30 years before it was declared a timberland.)This being the case, the
said possession has ripened into an ownership against the State, albeit an imperfect one.
 this should come under the meaning of "private rights" under Proclamation No. 2074 which are deemed segregated
from the mass of civil reservation granted to petitioner.

The central dispute is the interpretation of the first paragraph of Proclamation No. 2074:
“ I, FERDINAND E. MARCOS, President of the Philippines, do hereby set aside as Civil Reservation for Aklan National
College of Fisheries, subject to private rights, if any there be, parcels of land, containing an aggregate area of 24.0551
hectares, situated in the Municipality of New Washington, Province of Aklan, Philippines, “

The MCTC, the RTC and the Court of Appeals unanimously held that respondents retain private rights to the disputed
property, thus preventing the application of the above proclamation thereon. The private right referred to is an alleged
imperfect title, which respondents supposedly acquired by possession of the subject property, through their predecessors-
in-interest.
G.R. No. 199310, February 19, 2014

REPUBLIC OF THE PHILIPPINES, Petitioner, v. REMMAN ENTERPRISES, INC., REPRESENTED BY RONNIE P.


INOCENCIO, Respondent.

Facts: 

Remman Enterprises, Inc. (respondent), filed an application with the RTC for judicial confirmation of title over two parcels
of land situated in Barangay Napindan, Taguig, Metro Manila, identified as Lot Nos. 3068 and 3077, Mcadm–590–D, Taguig
Cadastre, with an area of 29,945 square meters and 20,357 sq.

The RTC called the case for initial hearing, only the Laguna Lake Development Authority (LLDA) appeared as oppositor. The
LLDA filed its Opposition to the respondent’s application for registration, alleged that the respondent’s application for
registration should be denied since the subject parcels of land are not part of the alienable and disposable lands of the
public domain; it pointed out that pursuant to Section 41(11) of Republic Act No. 4850 (R.A. No. 4850), lands, surrounding
the Laguna de Bay, located at and below the reglementary elevation of 12.50 meters are public lands which form part of
the bed of the said lake.

On the other hand, the Republic of the Philippines (petitioner), on July 16, 2002, likewise filed its Opposition, alleging that
the respondent failed to prove that it and its predecessors–in–interest have been in open, continuous, exclusive, and
notorious possession of the subject parcels of land since June 12, 1945 or earlier.

The respondent presented the following documents: (1) Deed of Absolute Sale dated August 28, 1989 executed by
Salvador and Mijares in favor of the respondent; (2) survey plans of the subject properties; (3) technical descriptions of the
subject properties; (4) Geodetic Engineer’s Certificate; (5) tax declarations of Lot Nos. 3068 and 3077 for 2002; and (6)
certifications dated December 17, 2002, issued by Corazon D. Calamno (Calamno), Senior Forest Management Specialist of
the DENR, attesting that Lot Nos. 3068 and 3077 form part of the alienable and disposable lands of the public domain. 

The RTC rendered a Decision, which granted the respondent’s application for registration of title to the subject properties. 

The RTC found that the respondent was able to prove that the subject properties form part of the alienable and disposable
lands of the public domain. The RTC opined that the elevations of the subject properties are very much higher than the
reglementary elevation of 12.50 m and, thus, not part of the bed of Laguna Lake.

The CA, by way of the assailed Decision,affirmed the RTC Decision dated May 16, 2007. The CA found that the respondent
was able to establish that the subject properties are part of the alienable and disposable lands of the public domain; that
the same are not part of the bed of Laguna Lake, as claimed by the petitioner.

Issue: Whether or not the respondent is entitled to the registration of title to the subject properties?

Ruling:

No. The subject properties are indeed part of the alienable and disposable lands of the public domain.

Section 14(1) of P.D. No. 1529 refers to the judicial confirmation of imperfect or incomplete titles to public land acquired
under Section 48(b) of Commonwealth Act (C.A.) No. 141, or the Public Land Act, as amended by P.D. No. 1073. Under
Section 14(1) of P.D. No. 1529, applicants for registration of title must sufficiently establish: first, that the subject land
forms part of the disposable and alienable lands of the public domain; second, that the applicant and his predecessors–in–
interest have been in open, continuous, exclusive, and notorious possession and occupation of the same; and third, that it
is under a bona fide claim of ownership since June 12, 1945, or earlier.

The first requirement was not satisfied in this case. To prove that the subject property forms part of the alienable and
disposable lands of the public domain, the respondent presented two certifications issued by Calamno, attesting that Lot
Nos. 3068 and 3077 form part of the alienable and disposable lands of the public domain. However, the said certifications
presented by the respondent are insufficient to prove that the subject properties are alienable and disposable.

In Republic of the Philippines v. T.A.N. Properties, Inc.,  the Court clarified that, in addition to the certification issued by
31

the proper government agency that a parcel of land is alienable and disposable, applicants for land registration must prove
that the DENR Secretary had approved the land classification and released the land of public domain as alienable and
disposable. They must present a copy of the original classification approved by the DENR Secretary and certified as true
copy by the legal custodian of the records.

The DENR certifications that were presented by the respondent in support of its application for registration are thus not
sufficient to prove that the subject properties are indeed classified by the DENR Secretary as alienable and disposable. It is
still imperative for the respondent to present a copy of the original classification approved by the DENR Secretary, which
must be certified by the legal custodian thereof as a true copy.

Anent the second and third requirements, the Court finds that the respondent failed to present sufficient evidence to
prove that it and its predecessors–in–interest have been in open, continuous, exclusive, and notorious possession and
occupation of the subject properties since June 12, 1945, or earlier.

The foregoing are but unsubstantiated and self–serving assertions of the possession and occupation of the subject
properties by the respondent and its predecessors–in–interest; they do not constitute the well–nigh incontrovertible
evidence of possession and occupation of the subject properties required by Section 14(1) of P.D. No. 1529. Indeed, other
than the testimony of Cerquena, the respondent failed to present any other evidence to prove the character of the
possession and occupation by it and its predecessors–in–interest of the subject properties.

Having failed to prove that the subject properties form part of the alienable and disposable lands of the public domain and
that it and its predecessors–in–interest have been in open, continuous, exclusive, and notorious possession and
occupation of the same since June 12, 1945, or earlier, the respondent’s application for registration should be denied.

GR No. 179155 April 2, 2014


NICOMEDES J. LOZADA, petitioner vs. EULALIA BRACEWELL ET AL., respondents
FACTS:
Petitioner Nicomedes Lozada filed an application for registration and confirmation of title over a parcel of land
covered by Plan PSU-129514, which was granted by the RTC-Makati City, acting as a land registration court. The
Land Registration Authority (LRA) consequently issued a Decree in the name of petitioner, who later obtained
Original Certificate of Title covering said parcel of land. Subsequently, respondent James Bracewell filed a petition
for review of the decree of registration under Section 32 of Presidential Decree No. 1529, otherwise known as the
“Property Registration Decree” before the RTC-Las Pinas City, claiming that a portion of the aforementioned Plan of
which he is the absolute owner and possessor, is fraudulently included in the earlier mentioned Decree. 
The RTC-Las Pinas City rendered a Decision finding that petitioner obtained the Decree in bad faith. Accordingly, it
directed the LRA to set aside said Decree and ordered petitioner to cause the amendment of the said Plan.
Aggrieved, petitioner elevated his case on appeal before the CA arguing mainly that the RTC-Las Pinas City had no
jurisdiction over a petition for review of a decree of registration under Section 32 of PD 1529, which should be filed
in the same branch of the court that rendered the decision and ordered the issuance of the decree.
The CA affirmed the decision of the RTC-Las Pinas, finding that respondents were able to substantiate their claim of
actual fraud in the procurement of said Decree, which is the only ground that may be invoked in a petition for review
of a decree of registration under Section 32 of PD No. 1529. It likewise held that, since the petition  for review was
filed within one (1) year from the issuance of the questioned decree and considering that the subject lot is located in
Las Pinas City, the RTC of said city had jurisdiction over the case.  With the motion for reconsideration denied,
petitioner thus sought relief via the instant petition for review challenging primarily the jurisdiction of the RTC-Las
Pinas City. 
ISSUE: 
Whether the RTC of Las Pinas City has jurisdiction over the petition for review of the Decree. 
RULING:
Yes, the RTC of Las Pinas City has jurisdiction over the petition for review of the Decree. 
Under Act No. 496 or the Land Registration Act, as amended, which was the governing law at the time of the
commencement by both parties of their respective registration proceedings, jurisdiction over all applications for
registration of title was conferred upon the RTCs of their respective provinces in which the land sought to be
registered is situated. The land registration laws were updated and codified under PD NO. 1529 and under Section 17
thereof, jurisdiction over an application for land registration is still vested on the RTC of the province or city where
the land is situated. Basically, Section 32 of PD No. 1529 provides that any person deprived of land or of any estate
or interest therein by such adjudication or confirmation of title obtained by actual fraud may file in the proper Court
of First Instance a petition for reopening and review of the decree of registration not later than one year from and
after the date of the entry of such decree of registration.    
Particularly, the Court refers to the fact that the application for original registration in this case was only filed before
the RTC of Makati City, Branch 134 because, during that time, i.e., December 1976, Las Piñas City had no RTC.
Barring this situation, the aforesaid application should not have been filed before the RTC of Makati City, Branch 134
pursuant to the rules on venue prevailing at that time. Under Section 2, Rule 4 of the 1964 Revised Rules of Court,
which took effect on January 1, 1964, the proper venue for real actions, such as an application for original
registration, lies with the CFI of the province where the property is situated, viz.:

Sec. 2. Venue in Courts of First Instance.— (a) Real actions. — Actions affecting title to, or for recovery of possession,
or for partition or condemnation of, or foreclosure of mortgage on, real property, shall be commenced and tried in
the province where the property or any part thereof lies.

As the land subject of this case is clearly situated in Las Pinas City, the application for its original registration should
have been filed before the RTC-Las Pinas City. 
WHEREFORE, the petition is DENIED. The Decision dated May 23, 2007 and the Resolution dated August 14, 2007 of
the Court of Appeals in CA-G.R. CV No. 81075 are hereby AFFIRMED.

 
VALIAO VS REPUBLIC OF THE PHILIPPINES
G.R. No. 170757       November 28, 2011

 
FACTS:
On August 11, 1987, petitioners Pacifico, Lodovico, Ricardo, Bienvenido Valiao, and Nemesio Grandea
filed with the RTC of Kabankalan, Negros Occ.l an application for registration of a parcel of land with an
area of 504,535 square meters, more or less, situated in Barrio Galicia, Municipality of Ilog, Negros
Occidental. However it was opposed the registration by the private oppositors Macario Zafra and Manuel
Yusay and the Republic of the Philippines, through the Office of the Solicitor General. 

In support of their claim of possession over the subject property, petitioners submitted in evidence Tax
Declaration No. 9562 dated September 29, 1976 under the names of the heirs of Basilio Millarez.

RTC granted petitioners' application for registration of the subject property. Private oppositors and the
Republic appeal with the CA.

CA reversed the trial court's findings and DECLARE the subject parcel of land to be inalienable and
indisposable land belonging to the public domain. Accordingto CA, the classification of lands of the
public domain is an exclusive prerogative of the executive department of the government and in the
absence of such classification, the lands remain as unclassified until it is released therefrom and
rendered open to disposition. Further, there exists a prior cadastral case involving the same parties
herein and the same Lot No. 2372, which ruled that Lot No. 2372 belongs to the Republic. 

 CA held that such judgment constitutes res judicata that bars a subsequent action for land
registration. It also ruled that the subject property is part of the inalienable land of the public domain and
petitioners failed to prove that they and their predecessors-in-interest had been in open, continuous,
exclusive and notorious possession of the land in question since June 12, 1945 or earlier
 
ISSUES:
1. WON the subject lot is alienable and disposable land of public domain. 
2. WON the alleged possession of the applicants through predecessors-in-interest is sufficient to sustain
their claim for prescription.

RULING:

1. No, the subject lot is not alienable and disposable land of public domain. 
Under the Regalian doctrine, all lands of the public domain belong to the State, which is the source of any
asserted right to any ownership of land. All lands not appearing to be clearly within private ownership are
presumed to belong to the State. Accordingly, public lands not shown to have been reclassified or
released as alienable agricultural land or alienated to a private person by the State remain part of the
inalienable public domain. Unless public land is shown to have been reclassified as alienable or
disposable to a private person by the State, it remains part of the inalienable public domain. To
overcome this presumption, incontrovertible evidence must be established that the land subject of
the application (or claim) is alienable or disposable.

Sec 14(par 1) of PD 1529 requires the petitioners to prove that: (1) the land forms part of the alienable and
disposable land of the public domain; and (2) they, by themselves or through their predecessors-in-interest,
have been in open, continuous, exclusive, and notorious possession and occupation of the subject land
under a bona fide claim of ownership from June 12, 1945 or earlier. 

These the petitioners must prove by no less than clear, positive and convincing evidence.

There must be a positive act declaring land of the public domain as alienable and disposable. To
prove that the land subject of an application for registration is alienable, the applicant must establish
the existence of a positive act of the government, such as a presidential proclamation or an
executive order; an administrative action; investigation reports of Bureau of Lands investigators;
and a legislative act or a statute. The applicant may also secure a certification from the government
that the land claimed to have been possessed for the required number of years is alienable and
disposable.

In the case at bar, no such evidence was offered by the petitioners to show that the land in question has
been classified as alienable and disposable land of the public domain. In the absence of incontrovertible
evidence to prove that the subject property is already classified as alienable and disposable, we must
consider the same as still inalienable public domain.

1. No, the alleged possession of the applicants through predecessors-in-interest is not sufficient to sustain their
claim for prescription.

2. The petitioners’ possession of the land in question from 1947 to 1966, petitioners could only support the same
with a tax declaration dated September 29, 1976.  At best, petitioners can only prove possession since said
date. What is required is open, exclusive, continuous and notorious possession by petitioners and their
predecessors-in-interest, under a bona fide claim of ownership, since June 12, 1945 or earlier. 
Petitioners failed to explain why, despite their claim that their predecessors-in-interest have possessed
the subject properties in the concept of an owner even before June 12, 1945, it was only in 1976 that they
started to declare the same for purposes of taxation. 

Moreover, tax declarations and receipts are not conclusive evidence of ownership or of the right to
possess land when not supported by any other evidence. The disputed property may have been
declared for taxation purposes in the names of the applicants for registration, or of their predecessors-in-
interest, but it does not necessarily prove ownership.  They are merely indicia of a claim of ownership.
G.R. No. 151312 August 30, 2006
HEIRS OF THE LATE SPOUSES PEDRO S. PALANCA AND SOTERRANEA RAFOLS VDA. DE
PALANCA namely: IMELDA R. PALANCA, MAMERTA R. PALANCA, OFELIA P. MIGUEL,
ESTEFANIA P. PE, CANDELARIA P. PUNZALAN, NICOLAS R. PALANCA, CONSTANTINO R.
PALANCA, EDMUNDO PALANCA, LEOCADIA R. PALANCA and OLIVERIO R. PALANCA,
represented by their attorney-in-fact, OFELIA P. MIGUEL, Petitioners,
vs.
REPUBLIC OF THE PHILIPPINES, (represented by the Lands Management Bureau), REGIONAL
TRIAL COURT OF PALAWAN (Office of the Executive Judge) and the REGISTER OF DEEDS OF
PALAWAN, Respondents.
 
FACTS:
In 1973, the heirs of Pedro S. Palanca, filed an application to bring the pieces of land allegedly owned
under the operation of the Land Registration Act. The Lands were situated in the Province of
Palawan. 
They inherited the lands from the late Pedro S. Palanca, who had occupied and possessed said land
openly and continuously in the concept of an owner since 1934, or 39 years before the filing of said
application. 
In Civil Case for "Recovery of Possession of a Parcel of Land", CFI of Palawan declared that the heirs
of Pedro S. Palanca, as the rightful possessors of the land. Jurisdictional requirements as to
notices, as prescribed by Section 31, Act No. 496, namely publication in the Official Gazette, were
complied with.
During the initial hearing of the case, verbal oppositions to the application were made by the Provincial
Fiscal of Palawan, in behalf of the Bureau of Forest Development, the Bureau of Lands, and the
Department of Agrarian Reform, some inhabitants of the subject properties and a businessman by the
name of Alfonso Guillamac. 
The Provincial Fiscal stated that the lands subject of the application had no clearance from the
Bureau of Forestry and that portions thereof may still be part of the timberland block and/or
public forest under the administration of the Bureau of Forestry and had not been certified as being
alienable and disposable by the Bureau of Lands. After 3 years from the date of the initial hearing,
no valid and formal opposition was filed by any of the oppositors in the form and manner required by
law to support his contention.

On the other hand, petitioners submitted the plan and technical description of the land, a survey
certificate approved by the Bureau of Lands and also tax declarations showing that they have
consistently paid the realty taxes accruing on the property. Petitioners likewise presented six
witnesses in support of their application.
 
Alfonso Lucero testified that he is a Forester in the Bureau of Forest Development, that he issued
certifications after due classification by his office, of alienable and disposable land for
administration by the Bureau of Lands and eventual disposition to interested parties.Therefore, the
area is no longer under the jurisdiction of the Bureau of Forest Development.

Alfonso Lucero also testified that as Chief of Land Classification Party No. 55, that the certifications he
issue carry much weight in land classification and releases in the province unless revoked by the
Manila Office.

Augustin O. Timbancaya, a licensed geodetic engineer, personally went to the lands in question,
executed the Plan of Lands which was also approved by the Director of Lands. The approval of the
Land Plans b the Director of the Bureau of Lands indicates that the lands in question have been
previously released for alienation and disposition.
CFI declares the petitioners as the owners in fee simple of the two parcels of land in
question.Original Certificate of Title (OCT) was issued in the name of petitioners. Subsequently, out of
OCT No. 4295, Transfer Certificates of Titles.

After 23 years, in year 2000, Republic of the Philippines filed with the CA a petition  for annulment of
judgment, cancellation of the decree of registration and title, and reversion, arguing that the decision
was null and void because the two lands in question were unclassified public forest land and, were not
capable of private appropriation. 

It presented Land Classification Map dated December 9, 1929 showing that the subject properties
were unclassified lands as well as a certification dated November 24, 2000 issued by the Community
Environment and Natural Resources Office stating that "the islands of Talampulan and Capar Island
located in the municipality of Busuanga, Palawan are within the unclassified public forest." 
Respondent likewise drew attention to Executive Proclamation No. 219 issued on July 2, 1967 which
classified the Province of Palawan as a National Game Refuge and Bird Sanctuary and the small
islands of Palawan as national reserves closed to exploitation and settlement under the administration
of the Parks and Wildlife Office, subject only to existing private rights. In view of the fact that the
properties were never classified as alienable and disposable.

In addition, respondent asserted that the participants in the proceedings committed perfidious acts
amounting to extrinsic fraud which is one of the grounds for the annulment of a judgment. It says that
culture of collusion existed between and among the petitioners. 

CA declared CFI decision NULL and VOID. The decree and the corresponding OCT issued in the
name of the Heirs of Pedro S. Palanca, as well as the subsequent TCTs and all subsequent TCTs
issued thereafter are also declared NULL and VOID. 

Petitioners’ contention: 
CA disregarded settled jurisprudence and applicable land laws when it ruled that the subject properties
were forest lands and that, consequently, the land registration court did not have jurisdiction to award
the same to them. 

It is not necessary for them to prove that the government had expressly given a grant of the subject
properties to Pedro S. Palanca, separate of the legislative grant given to them purportedly under
Commonwealth Act No. 141 (Public Land Act). 
Particular land need not be formally released by an act of the Executive before it can be deemed open
to private ownership.
CA erred in relying upon Executive Proclamation No. 219 and upon Land Classification Map No. 839,
Project 2-A to nullify petitioners’ mother title. 
The reversal of the CFI’s decision violated the principle of res judicata as well as the rule on
incontrovertibility of land titles under Act No. 496.
 
Respondent’s contends that 
A. the claim that the subject parcels of land are public agricultural lands by virtue of a legislative grant
is unfounded and baseless; 
B. the land registration court of Puerto Princesa, Palawan, was devoid of jurisdictional competence to
order titling of a portion of forest land; 
C. the CA is correct in declaring that there must be a prior release of the subject lands for agricultural
purposes; 
D. the rules on res judicata and the incontestability of Torrens titles do not find proper applications in
the exercise of the power of reversion by the State; and 
E. estoppel and laches will not operate against the State.
F.  Respondent also reiterates its contention that collusion existed between the parties in the
proceedings below which prevented a fair submission of the controversy, to the damage and prejudice
of the Republic.
 
ISSUES:
I. WON there is necessity for the applicant to prove that the government had expressly given a
grant of the subject properties.

Yes, In an action to annul a judgment, the burden of proving the judgment’s nullity rests upon the
petitioner. The petitioner has to establish by clear and convincing evidence that the judgment being
challenged is fatally defective. 

Section 48(b) of the Public Land Act states that:


Sec. 48. The following-described citizens of the Philippines, occupying lands of the public
domain or claiming to own any such lands or an interest therein, but whose titles have not been
perfected or completed, may apply to the Court of First Instance of the province where the land is
located for confirmation of their claims and the issuance of a certificate of title therefor, under the
Land Registration Act, to wit:
xxx
(b) Those who, by themselves or through their predecessors-in-interest, have been in
continuous, exclusive, and notorious possession and occupation of agricultural lands of the
public domain, under a bona fide claim of acquisition or ownership, for at least thirty years
immediately preceding the filing of the application for confirmation of title, except when prevented
by war or force majeure. Those shall be conclusively presumed to have performed all the
conditions essential to a government grant and shall be entitled to a certificate of title under the
provisions of this chapter.
The above provision clearly requires the concurrence of two things: (1) that the land
sought to be registered is public agricultural land, and (2) that the applicant seeking
registration must have possessed and occupied the same for at least thirty years prior to the
filing of the application. In this case, what is in doubt is the compliance with the first requisite.
Burden of the State to prove that the land which it avers to be of public domain is really of such nature
applies only in instances where the applicant has been in possession of the property since time
immemorial. When referring to this type of possession, it means possession of which no person living
has seen the beginning and the existence of which such person has learned from the latter’s elders for
it justifies the presumption that the land had never been part of the public domain or that it had been
private property even before the Spanish conquest. The possession of petitioners in this case does not
fall under the above-named exception as their possession, by their own admission, only commenced
sometime in 1934.

Where there is a showing that lots sought to be registered are part of the public domain, the applicant
for land registration under Section 48 of CA No. 141 must secure a certification from the
government that the lands claimed to have been possessed by the applicant as owner for more
than 30 years are alienable and disposable. Petitioners’ failure to do so in this case, when taken
with the evidence adduced by respondent showing that the lands in question indeed remain part of the
public domain and form part of the national reserves, confirms that the CFI never acquired jurisdiction
to order the registration of such lands in favor of petitioners, and certainly justifies their reversion to the
State.
 

I. WON particular land need not be formally released by an act of the Executive before
it can be deemed open to private ownership.

Yes. Pursuant to Constitutional precepts, all lands of the public domain belong to the State, and the
State is the source of any asserted right to ownership in such lands and is charged with the
conservation of such patrimony.  Thus, the Court has emphasized the need to show in registration
proceedings that the government, through a positive act, has declassified inalienable public land into
disposable land for agricultural or other purposes. 

Registration of the properties is sought under Commonwealth Act No. 141. Sections 6 and 7 of the Act
provide as follows:
Section 6. The President, upon the recommendation of the Secretary of Agriculture and
Commerce, shall from time to time classify the lands of the public domain into —
(a) Alienable or disposable,
(b) Timber, and
(c) Mineral lands,
and may at any time and in a like manner transfer such lands from one class to another, for the
purposes of their administration and disposition.

Section 7. For the purposes of the administration and disposition of alienable or disposable public
lands, the President, upon recommendation by the Secretary of Agriculture and Commerce, shall from
time to time declare what lands are open to disposition or concession under this Act.

Based on the foregoing, the classification or reclassification of public lands into alienable or
disposable, mineral or forest lands is the exclusive prerogative of the Executive Department of
the government. Clearly, the courts no longer have the authority, whether express or implied, to
determine the classification of lands of the public domain. 
 
III WON the subject parcels of land are public agricultural lands by virtue of a legislative grant
No, as stated earlier, the classification or reclassification of public lands into alienable or
disposable, mineral or forest lands is the exclusive prerogative of the Executive Department of
the government and NOT by Legislative Department
 
IV. WON CA erred in relying upon Executive Proclamation No. 219 and upon Land
Classification Map No. 839, Project 2-A to nullify petitioners’ mother title. 
NO. Land Classification Map  indicates that the Talampulan and Capari Islands on which the
properties were located were unclassified public lands. And by virtue of Executive Proclamation No.
219 these islands were subsequently classified as national reserves. Based on these, it becomes
evident that the subject properties have never been released for public disposition. These properties
remained as inalienable public lands.

While it is true that the land classification map does not categorically state that the islands are public
forests, absence of the classification as mineral or timber land, the land remains unclassified land until
released and rendered open to disposition. When the property is still unclassified, whatever
possession applicants may have had, and however long, still cannot ripen into private ownership.
 
V. WON The reversal of the CFI’s decision violated the principle of res judicata as well as the
rule on incontrovertibility of land titles under Act No. 496.
No, non-disposable public lands registered under the Land Registration Act may be
recovered by the State at any time and the defense of res judicata would not apply as courts
have no jurisdiction to dispose of such lands of the public domain. 
 
VI. WON estoppel and laches will not operate against the State
No, an action for reversion filed by the State to recover property registered in favor of
any party which is part of the public forest or of a forest reservation never prescribes. 
 
 
 Final Conclusion:
To reiterate, the validity of the CFI decision was impugned on the basis of the court’s lack of
jurisdiction. If the properties were alienable public lands, then the CFI, acting as a land registration
court, had jurisdiction over them and could validly confirm petitioners’ imperfect title. Otherwise, if the
properties were indeed public forests, then the CA was correct in declaring that the land registration
court never acquired jurisdiction over the subject matter of the case and, as a result, its decision
decreeing the registration of the properties in favor of petitioners would be null and void.
The reason for this is the fact that public forests are inalienable public lands. The possession of public
forests on the part of the claimant, however long, cannot convert the same into private
property.  Possession in such an event, even if spanning decades or centuries, could never ripen into
ownership.  It bears stressing that unless and until the land classified as forest is released in an official
proclamation to that effect so that it may form part of the disposable lands of the public domain, the
rules on confirmation of imperfect title do not apply. 
 
Laureano Hermoso vs Court of Appeals 
The present case is a petition for review on Certiorari under Rule 45 of the Rule of court, assailing the judgment
of Oct. 15, 2004 and the resolution dated Jan. 19, 2005. 
The case involves parcels of land located in Malhacan, Meycauayan, Bulacan, identified as Lot 3257 owned by
Petra Francia and Lot 3415 owned by Antonio Francia. Which comprises an area of 2.5 and 1.5850 Hectares
respectively. Both lots form part of a larger parcel of land with an area of 32.1324 hectares owned by Amos Jr.,
Benjamin, Cecilia, Petra, Antonio and Rufo, all surnamed Francia. 
Since 1978, Laureano Hermoso and Miguel Banag have been occupying and cultivating the aforementioned
lots as tenants thereof. They filed a petition for coverage of lots under Presidential Decree no. 27, and on July 4,
1995, the Department of Agrarian Reform issued an order granting both Hermoso and Banag the respective
portions of land. 
Respondents Heirs of Antonio and Petra Francia filed an omnibus motion for reconsideration and
reinvestigation, to which DAR affirmed with modification the earlier order and disposed of the case at hand. 
In a separate case, Hermoso and Banag filed with DAR Adjuidcation Board(DARAB), the case delving on
whether Hermoso and Banag were tenants of Heirs of Francia in subject holding. On June 13, 1996, DARAB
upheld the tenancy relationship of Hermoso and banag with Heirs of Francia.
Heirs of Francia filed a motion for reconsideration, but the same was denied. Petition for review on certiorari
and reconsideration was filed before the CA, but both were denied.
The case was elevated to the supreme court, but the court denied the petition for lack of verification, and
subsequently also denied the motion for reconsideration in a resolution. 
Earlier in January 20, 1997, Banag filed before DAR an urgent exparte motion for issuance of emancipation
patent, to which DAR granted the motion. On March 21, Heirs of Francia filed a motion for reconsideration,
claiming that the lands involved have been approved for conversion to urban purposes in an order issued by
the DAR secretary. DAR issued an order affirming the order granting the emancipation patent in favor of Banag,
to which the office of the President, through it’s deputy Executive Secretary rendered a decision dismissing the
case at hand, and affirming the emancipation patent in favor of Banag. 
Heirs of Francia filed with the CA a petition for review, stating that PD 27 does not cover the subject parcels of
land pursuant to the June 1973 order of DAR secretary reclassifying the lands and declaring the same as
suited for residential, commercial, industrial or other purposes. Furthermore, Housing Land and Use
Regulatory Board(HLURB) reclassified the lands as early as Oct 14, 1978. 
CA ruled in favor of Heirs of Francia, reversing the decision of the Office of the President, and granting the
instant petition. 
Hermoso filed a motion for reconsideration with the CA, but the appellate court denied the motion. Hence, the
present petition. 
Issue: WON Lots 3257 and 3415 are covered PD 27. 
Ruling: Lots 3257 and 3415 are not covered by PD 27. 
For the parcels of land subject of this petition to come within the coverage of P.D. No. 27, it is necessary to
determine whether the land is agricultural. Section 3(c) of R.A. No. 6657 defines agricultural land, as
follows:
(c) Agricultural Land refers to the land devoted to agricultural activity as defined in this Act and not
classified as mineral, forest, residential, commercial or industrial land.
and Section 3(b) specifies agricultural activity as:
(b) Agriculture, Agriculture Enterprise or Agricultural Activity means cultivation of the soil, planting of
crops, growing of fruit trees, including the harvesting of such farm products, and other farm
activities and practices performed by a farmer in conjunction with such farming operations done by
persons whether natural or juridical.
On the basis of these definitions, the subject parcels of land cannot be considered as within the ambit of
P.D. No. 27. This considering that the subject lots were reclassified by the DAR Secretary as suited for
residential, commercial, industrial or other urban purposes way before petitioner filed a petition for
emancipation under P.D. No. 27. The pertinent portions of the June 5, 1973 Order  read: 25

Pursuant to the provisions of Republic Act 3844, as amended, the said requests of the petitioners were
referred to the National Planning Commission as well as to the Agrarian Reform Team Leader, Valenzuela,
Bulacan for proper investigation.

The National Planning Commission in compliance therewith after due investigation and physical survey
of the subject areas, favorably recommended the suitability of the same to residential, commercial,
industrial or other urban purposes.

Similarly, the Agrarian Reform Team in Valenzuela, Bulacan after due investigation thereof found the
parcels of land subject hereof highly suitable for conversion into urban purposes in view of his findings and
verification of the location, facilities necessary for urban development and also, the low agricultural income
thereof (unirrigated), of the said land. The Team Leader concerned in his recommendation submitted to this
Office made mentioned (sic) that in his declaration of the suitability of the subject properties for urban
purposes, he believes that the conformity of the tenants consisting of eleven (11) tenants are no longer
needed so long as the petitioners are willing to pay the disturbance compensation as provided for
by law.

The petitioners manifested to the Team Leader concerned their willingness to pay each and every tenant
the disturbance compensation according to law. To show further their sincerity to comply with the provisions
of the law on disturbance compensation, and to show that their (petitioners) purpose of the instant request
is not to evade the provisions of Decree 27, they stated in their letter-request that they will not eject any
tenants therefrom, nor dispossessed (sic) them of their landholdings until after they are fully and justly paid
the disturbance compensation according to law.

The subject parcels of land are not included in the land transfer operation according to the team’s
report.It maybe mentioned in this connection, that from the report of the National Planning Commission
submitted to this Office, it appears that the subject properties are strategically located in the urban
center of the town of Meycauayan wherein there are already existing developed and occupied residential
subdivisions and even low cost housing projects subsidized by funds from government financial institution.
Likewise, there are also industrial establishments in its vicinity according to the National Planning
Commission’s report.

In view of the foregoing, and considering the parcels of land subject hereof to be suited for residential,
commercial, industrial or other urban purposes as found and recommended by the National Planning
Commission and the Agrarian Reform Team concerned, and considering further that the said parcels of
land by reason of their location and the existence of developed and occupied residential subdivisions and
industrial establishments in the immediate vicinity maybe considered as one of the possible areas to be
reserved for urban development as contemplated in the Letter of Instruction No. 46 of the President,
and considering finally, that the right of the agricultural tenants therein will be fully compensated and
there will be no ejectment of tenants until after full payment thereof, as manifested by the petitioners,
the instant requests of the petitioners should be, as hereby it is, given due course and the parcels of land
subject thereof are hereby declared suited for residential, commercial, industrial or other urban purposes in
accordance with the provisions of Republic Act 3844, as amended.
It is understood however, that no agricultural tenants and/or lessees shall be ejected from or dispossessed
of their landholdings by virtue of this Order not until after they are duly and justly paid the disturbance
compensation according to law, the amount of which maybe determined and fixed by the proper court in the
absence of any mutual agreement thereto by and between the agricultural lessees and the owner-
petitioners.
 
 

Secretary of the DENR v Yap 


G.R. 172775 

There are two consolidated petitions. 


The first is G.R. No. 167707, a petition for review on certiorari of the Decision of the Court of Appeals affirming that of the
Regional Trial Court in Kalibo, Aklan, which granted the petition for declaratory relief filed by respondents-claimants Mayor
Jose Yap, et al. and ordered the survey of Boracay for titling purposes.
The second is G.R. No. 173775, a petition for prohibition, mandamus, and nullification of proclamation No. 1064 issued by
President Gloria Macapagal-Arroyo classifying Boracay into reserved forest and agricultural land. 

Facts: 

GR no. 167707 
On April of 1976, the DENR approved the National reservation survey of Boracay which identified several lots as being
occupied by named persons. On November of 1978, Pres. Marcos issued PP 1801 declaring Boracay Island as tourist zones
and marine reserves under the administration of the Philippine Tourism Authority (PTA), then later on approves PTA Circ.
3-82 to implement the said proclamation. 

Yap, together with other respondents filed a petition for declaratory relief with the RTC of Kalibo, Aklan claiming that the
said proclamation prevented them from filing an application for judicial confirmation of imperfect title or survey of land for
titling purposes. Moreover, they contend that they or through their predecessor-in interest has been in an open,
continuous, exclusive and notorious possession of the subject land since time immemorial and they were paying their
realty tax. 

Further, the said proclamation did not place Boracay beyond the commerce of men, classified as a tourist zone,
therefore, susceptible of private ownership. Invoking Sec 48(b) of CA 141 otherwise known as the Public Land Act, they
have the right to have the lots registered under their names. 

The republic, through OSG, opposed the petition of the respondents’ claiming that Boracay Island was an unclassified land
of public domain. Invoking Sec 3(a) or PD 705 or the revised forestry code, as amended, it is not susceptible of private
ownership. The OSG maintained that the right of the respondent to judicial confirmation of title was governed by PD 705
and CA 141. Since the Island has not been classified as alienable and disposable, whatever possession they cannot ripen
into ownership. 

RTC: It upheld respondents’ right to have their occupied lands titled in their name. It ruled that neither Proclamation No.
1801 nor PTA Circular No. 3-82 mentioned that lands in Boracay were inalienable. The Circular itself recognized private
ownership of lands. The trial court cited Sections 87 and 53 of the Public Land Act as basis for acknowledging private
ownership of lands in Boracay and that only those forested areas in public lands were declared as part of the forest
reserve.

OSG moved for reconsideration but it was denied. 


Thus, they appealed it to the CA. 

CA: Affirmed the decision of the lower court. 

OSG moved for reconsideration, but, it was also denied. 


They appeled to the SC. 
GR no. 173775 
However, during the pendency of the case (GR no. 167707) Pres. Arroyo issued PP 1064 which classifies Boracay into 400
ha of forest land and 628.96 ha or agricultural land. On august of 2006, Sacay, Gelito and other land owners filed a petition
for nullification of PP1064. They contend that the said proclamation infringes their “prior vested right” over portions of
Boracay, also, they have been occupying the lot since time immemorial. 

Nonetheless, the OSG argued that they don’t have vested rights over the property because Boracay is an unclassified
public forest land ursuant to PD 705. Therefore, they are neither alienable nor disposable lands. 

On November of 2006, the court ordered the consolidation of both cases as they involve the same issues on land
classification of Boracay Island.

Issue: 
Whether or not private claimants have a right to secure titles over their occupied portions in Boracay. 

Ruling: No

The Court of Appeal’s decision was reversed. 


Except for lands already covered by existing titles, Boracay was an unclassified land of the public domain prior to
Proclamation No. 1064. Such unclassified lands are considered public forest under PD No. 705. PD No. 705 issued by
President Marcos categorized all unclassified lands of the public domain as public forest. Appying this law, all unclassified
lands, including that of the Boracay Island is considered as Public forest. Nonetheless, PD No. 705 respects titles already
existing prior to its effectivity. 

The 1935 Constitution classified lands of the public domain into agricultural, forest or timber, such classification modified
by the 1973 Constitution. The 1987 Constitution reverted to the 1935 Constitution classification with one addition:
national parks. Of these, only agricultural lands may be alienated. Prior to Proclamation No. 1064 of May 22, 2006, Boracay
Island had never been expressly 
and administratively classified under any of these grand divisions. Boracay was an unclassified land of the public domain. 

A positive act declaring land as alienable and disposable is required. In keeping with the presumption of State ownership,
the Court has time and again emphasized that there must be a positive act of the 
government, such as a presidential proclamation or an executive order; an administrative action; investigation reports of
Bureau of Lands investigators; and a legislative act or a statute. In fact, Section 8 of CA No. 141 limits alienable or
disposable lands only to those lands which have been "officially delimited and classified." The applicant may also secure a
certification from the government that the land claimed to have been possessed for the required number of years is
alienable and disposable. The burden of proof in overcoming the presumption of State ownership of the lands of the public
domain is on the person applying for registration (or claiming ownership). 

In the case at bar, no such proclamation, executive order, administrative action, report, statute, or certification was
presented to the Court. The records are bereft of evidence showing that, prior to 2006, the portions of Boracay occupied
by private claimants were subject of a government proclamation that the land is alienable and disposable. Absent such
well-nigh incontrovertible evidence, the Court cannot accept the submission that lands occupied by private claimants were
already open to disposition before 2006. Matters of land classification or reclassification cannot be assumed. They call for
proof.

Proc. No. 1801 cannot be deemed the positive act needed to classify Boracay Island as alienable and disposable land. If
President Marcos intended to classify the island as alienable and disposable or forest, or both, he would have identified
the specific limits of each, as President Arroyo did in Proclamation No. 1064. This was not done in Proclamation No. 1801. 

Noting that millions were invested for the development of Boracay Island making internationally known ad some called it
their home. However, the SC expressed that they are bound to apply the law 
strictly and judiciously. Thus, saying “Ito and batas, ito ang dapat umiral.” 
G.R. No. 135385               December 6, 2000
ISAGANI CRUZ and CESAR EUROPA, petitioners,
vs.
SECRETARY OF ENVIRONMENT AND NATURAL RESOURCES, et.al.
Facts:

Petitioners Isagani Cruz and Cesar Europa brought this suit for prohibition and mandamus as citizens and
taxpayers, assailing the constitutionality of certain provisions of Republic Act No. 8371 (R.A. 8371), otherwise
known as the Indigenous Peoples Rights Act of 1997 (IPRA), and its Implementing Rules and Regulations
(Implementing Rules).
Petitioners assail the constitutionality of the following provisions of the IPRA and its Implementing Rules on the
ground that they amount to an unlawful deprivation of the State’s ownership over lands of the public domain
as well as minerals and other natural resources therein, in violation of the regalian doctrine embodied in
Section 2, Article XII of the Constitution.
Petitioners also content that, by providing for an all-encompassing definition of "ancestral domains" and
"ancestral lands" which might even include private lands found within said areas, Sections 3(a) and 3(b) violate
the rights of private landowners. 3

In addition, petitioners question the provisions of the IPRA defining the powers and jurisdiction of the NCIP
(National Commission on Indigenous Peoples) and making customary law applicable to the settlement of
disputes involving ancestral domains and ancestral lands on the ground that these provisions violate the due
process clause of the Constitution.
Respondents Secretary of the DENR and Secretary of the Department of Budget and Management (DBM) filed
through the Solicitor General a consolidated Comment. The Solicitor General is of the view that the IPRA is
partly unconstitutional on the ground that it grants ownership over natural resources to indigenous peoples
and prays that the petition be granted in part.
Issue: Issue: Whether or not the IPRA law (specifically the aforementioned sections therein) is unconstitutional.
Ruling: NO. After due deliberation on the petition,  7 members of the court voted to dismiss the petition.
No, the provisions of IPRA do not contravene the Constitution. Examining the IPRA, there is nothing in the law
that grants to the ICCs/IPs ownership over the natural resources within their ancestral domain. Ownership over
the natural resources in the ancestral domains remains with the State and the rights granted by the IPRA to the
ICCs/IPs over the natural resources in their ancestral domains merely gives them, as owners and occupants of
the land on which the resources are found, the right to the small scale utilization of these resources, and at the
same time, a priority in their large scale development and exploitation.
Additionally, ancestral lands and ancestral domains are not part of the lands of the public domain. They are
private lands and belong to the ICCs/IPs by native title, which is a concept of private land title that existed
irrespective of any royal grant from the State. However, the right of ownership and possession by the ICCs/IPs
of their ancestral domains is a limited form of ownership and does not include the right to alienate the same. 

Summary of separate opinions of each Justice


Q1: Whether or not Sections 3(a)(b), 5, 6, 7, 8, 57 and 58 of IPRA unlawfully deprive the state of its ownership
of lands of public domain and violate the regalian doctrine.
Justice Kapunan: NO. Said provisions affirming the IPs’ ownership of their ancestral land by virtue of native title
do not diminish the state’s ownership of lands within public domain because ancestral lands and domains are
considered private land.
Sec. 3(a) does not confer any right of ownership over the natural resources to the ICCs/IPs. Its purpose is
merely definitional. Sec. 57 only grants priority rights to ICCs/IPs in the utilization of natural resources and not
absolute ownership thereof.
Justice Puno: NO. Ancestral lands and domains are not part of public domain. They are private and belong to
the ICCs/IPs. The classification of lands in the public domain under Sec. 3, Art. XII of the
Constitution does not include ancestral lands nor domain.
The rights of ICCs/IPs to their ancestral domains and lands may be acquired in 2 modes: (1) native title or (2)
torrens title under the Public Land Act and Land Registration Act with respect to ancestral lands only.
Both modes recognize the land as private. However, by including “natural resources”, Sec. 1, Part II, Rule III of
Implementing Rules goes beyond Sec. 7(a) and therefore is unconstitutional.
Justice Panganiban: YES. Sec. 3(a) contravenes Sec. 2, Art. XII of the Constitution. IPRA does not specify the
limits to ancestral lands and domains. It also relinquishes the state power of full control of natural resources in
ancestral lands and domains in favor of ICCs/IPs who may exercise these rights without any time limit.
Justice Vitug: YES. Sec. 7 and 57 go beyond the context of the fundamental law.
Republic of the Philippines, petitioner
versus
Celestina Naguiat, respondent
G.R. No. 134209. January 24, 2006

Facts:

Celestina Naguiat, an Filipino citizen, resident of Angeles City Pampanga, applied for
registration before the Regional Trial Court of Zambales, four parcels of land located in
Panan, Botolan Zambales.

She alleges to be the owner of the said parcels of land having acquired them by
purchase from an entity who have been in possession thereof for more than thirty (30) years
that had not suffered from any mortgage or encumbrance of whatever kind nor is there any
person having any interest, legal or equitable, or in possession thereof.

The Republic of the Philippines filed an opposition to the application assailing the
ownership of the respondent on the grounds of open, continuous, exclusive and notorious
possession and occupation of the lands in question by the predecessors-in-interest that is
an ownership in fee simple on the basis of Spanish title or grant that is no longer
applicable; the subject properties are part of the public domain belonging to the Republic
of the Philippines not subject to private appropriation.

The trial court adjudicated the land to Naguiat and affirmed by the appellate court,
hence the petition for review.

Petitioners Theory:
The theory of [petitioner] that the properties in question are lands of the public domain
cannot be sustained as it is directly against the above doctrine. Said doctrine is a
reaffirmation of the principle established in the earlier cases . . . that open, exclusive and
undisputed possession of alienable public land for period prescribed by law creates the legal
fiction whereby the land, upon completion of the requisite period, ipso jure and without the
need of judicial or other sanction, ceases to be public land and becomes private property.

Issue:

Whether the areas in question still part of public domain?

Ruling:

The Court ruled that the areas are still part of the public domain. The respondent failed to
present the required certification from the proper government agency or
proclamation reclassifying the land applied for as alienable and disposable.

Under Section 2, Article XII of the Constitution, under the Regalian doctrine, all lands of the
public domain belongs to the State. Accordingly public lands not shown to have been
reclassified or released as alienable agricultural land or alienated to a private person by the
State remain part of the inalienable public domain. 
Anent thereto, under the Section 6 of the Public Land Act, the prerogative of classifying and
reclassifying lands of the public domain belongs to the Executive Branch and not with the
court. The lower court and the appellate court erred in deciding in favor of the respondent
with subsisting matters that rest in the powers of the Executive Branch of government.

Unclassified land cannot be acquired by adverse occupation or possession


unless until determined by the proper government agency or proclamation
reclassifying the land agricultural thus alienable and disposable. The Court properly
REVERSED and SET ASIDE the decisions of the lower court and appellate court and the
application of the respondent DENIED.

REPUBLIC VS AGUNOY

G.R. No. 155394           February 17, 2005

FACTS:

Agunoy filed his application for a free patent covering two parcels of land. In 1967, the Director of
Lands issued Agunoy’s patent and such was subsequently registered in the Register of Deeds with
an issuance of the corresponding OCT.

The same year, the heirs of one Eusebio Perez caused the annotation on the OCT of an adverse
claim over a portion of the subject lots. They also filed a formal protest, alleging that the one of the
lots had been adjudicated as their private property pursuant to a CFI decision in 1960.

The Bureau of Lands investigated and ascertained that the patent and OCT in Agunoy’s favor were
improperly and fraudulently issued.

Later, however, the heirs of Agunoy executed a deed of extrajudicial partition with sale in favor of 
one Joaquin Sangabol. Sangabol was issued a TCT and later sold portions of the property to
Sps Dee. The annotated adverse claim of the heirs of Perez was cancelled by the ROD.

In 1988, heirs of Perez filed a supplemental protest alleging that they and their predecessors have
exclusively occupied and cultivated the subject lands; that Agunoy never occupied nor cultivated the
land; and that the patent and title issued to the latter were obtained through fraud and
misrepresentation.
During these proceedings, the Republic filed the complaint in this case against Agunoy and his
heirs, as well as the Sps Dee, alleging: 1) that the patent and the OCT in favor of Agunoy were
procured through fraud, deceit, and misrepresentation since the property was already adjudicated as
private property of the heirs of Perez; and 2) the subject property was no longer a disposable
public land.

RTC ruled in favor of the Republic and declared the patent and OCT, as well as other subsequent
TCTs emanating therefrom as null and void.

The CA, however, reversed and declared that the Republic is not the real party-in-interest in this
case. Hence, this recourse by the Republic.

ISSUES:
1. WON the Republic is a real party-in-interest 
2. WON Sps Dee acquired the title over a portion of the subject property for value and in good faith

RULING:

1. No, The Republic is not the real party-in-interest.

In the law of procedure, every action must be prosecuted or defended in the name of the real party-in-
interest, meaning, ‘the party who stands to be benefited or injured by the judgment in the suit, or the
party entitled to the avails of the suit.’

Consequently, the Republic is not a real party-in-interest and it may not institute the instant
action. With its very admission in its pleading that the subject property are already private
properties of the heirs of Perez and are therefore, no longer disposable public land, it cannot
still profess to be the real party-in-interest. As no less admitted by the Republic, the lands
subject are no longer part of the public domain and the nullification of Agunoy’s patent and
OCT would not result in the reversion of the lands to the mass of public land.

2. Yes, After sleeping for an unreasonably long period of time lasting for decades, the heirs of Eusebio
Perez can longer defeat the better right arising from the Torrens titles in the names of the present
transferees of the properties, unless and until anyone succeeds in overcoming the presumption of
good faith in securing their respective titles.
3. The real purpose of the Torrens System is to quiet title to land and stop forever any question as to
its legality. Once a title is registered, the owner may rest secure without the necessity of waiting in the
portals of the court. Titles over lands under the Torrens System should be given stability for on it
depends the stability of the country’s economy.

 
 

Leticia P. Ligon vs Court of Appeals 


Facts: 
On October 1990 respondent Iglesia ni Kristo(INK) filed with the Regional Trial Court a claim for
specific specific performation with damages against the Islamic Directorate of the Philippines.
INK alleged in it’s complaint that by virtue of an Absolute Deed of Sale IDP sold to the former 2 parcels
of land located at Tandang Sora, Barrio Culiat, Quezon City, both of which IDP is the registered
owner. The parties stipulated in the deed of sale that the IDP shall undertake to evict all squatters
and illegal occupants in the property within 45 days from the execution of the contract. 
IDP failed to fulfill it’s obligation hence INK prayed that the trial court order IDP to comply with it’s
obligation of clearing the subject lots of illegal occupants and to pay damages to INK. 
INK filed a motion for partial summary judgment on the ground that there was actually no genuine
issue as to any material fact. The trial court rendered partial judgment, and on October 1991 an
amended partial judgment granting the reliefs prayed for by INK except the prayer for damages
which was to be resolved later. 
On January 1992 INK filed a motion in the same case praying that petitioner Leticia Tigon, who
was in possession of the certificate of title over the properties as mortgagee of IDP, be directed
to surrender the certificate to the Register of Deeds of Quezon City for the registration of the
Absolute Deed of Sale in it’s name.  
On January 1992 Ligon filed an opposition to the motion on the ground that IBP was not served
copy of the motion, and the ownership of the INK over the property was still in issue since rescission
was sought by the IDP as a counterclaim. 
On February 1992 Ligon filed a supplemental opposition questioning the jurisdiction of the trial
court because the motion involved the registrability of the document of sale, and she was not made a
party to the main case. 
The trial court granted the motion of the INK and ordered Ligon to surrender to the INK the
owner’s copy of RT-26521 (170567) and RT-26520 (176616) in open court for the registration of the
Absolute Deed of Sale in the latter's name and the annotation of the mortgage executed in favor of
petitioner on the new transfer certificates of title to be issued to INK. 

On 6 April 1992, on motion of petitioner Ligon, the trial court reconsidered its order by directing her to
deliver the certificates of title to the Register of Deeds of Quezon City.  3

Ligon filed a petition for certiorari with the Court of Appeals seeking the annulment of the two (2)
orders. However, on 28 October 1992 the Court of Appeals dismissed the petition and affirmed
the orders of the trial court.

Issue: WON ligon is entitled to registration of ownership of the land in question. 


Ruling: no, ligon Is not entitled to the land in question. 
Under our land registration law, no voluntary instrument shall be registered by the Register of Deeds unless
the owner's duplicate certificate is presented together with such instrument, except in some cases or upon
order of the court for cause shown. In case the person in possession of the duplicate certificates refuses or
fails to surrender the same to the Register of Deeds so that a voluntary document may be registered and a
new certificate issued, Sec. 107, Chapter 10, of P.D. No. 1529 clearly states:
Sec. 107. Surrender of withheld duplicate certificates. — Where it is necessary to issue a
new certificate of title pursuant to any involuntary instrument which divests the title of the
registered owner against his consent or where a voluntary instrument cannot be registered
by reason of the refusal or failure of the holder to surrender the owner's duplicate certificate
of title, the party in interest may file a petition in court to compel surrender of the
same to the Register of Deeds. The court, after hearing, may order the registered owner
or any person withholding the duplicate certificate to surrender the same and direct the
entry of a new certificate or memorandum upon such surrender. If the person withholding
the duplicate certificate is not amenable to the process of the court, or if for any reason the
outstanding owner's duplicate certificate cannot be delivered, the court may order the
annulment of the same as well as the issuance of a new certificate of title in lieu
thereof. Such new, certificate and all duplicates thereof shall contain a memorandum of the
annulment of the outstanding duplicate.
Before the enactment of P.D. No. 1529 otherwise known as the Property Registration Decree, the former
law, Act No. 496 otherwise known as the Land Registration Act, and all jurisprudence interpreting the
former law had established that summary reliefs such as an action to compel the surrender of owner's
duplicate certificate of title to the Register of Deeds could only be filed with and granted by the Regional
Trial Court sitting as a land registration court if there was unanimity among the parties or there was no
adverse claim or serious objection on the part of any party in interest, otherwise, if the case became
contentious and controversial it should be threshed out in an ordinary action or in the case where the
incident properly belonged. 4

Under Sec. 2 of P.D. No. 1529, it is now provided that "Courts of First Instance (now Regional Trial Courts)
shall have exclusive jurisdiction over all applications for original registration of titles to lands, including
improvements and interest therein and over all petitions filed after original registration of title, with power to
hear and determine all questions arising upon such applications or petitions." The above provision has
eliminated the distinction between the general jurisdiction vested in the regional trial court and the limited
jurisdiction conferred upon it by the former law when acting merely as a cadastral court. Aimed at avoiding
multiplicity of suits the change has simplified registration proceedings by conferring upon the regional trial
courts the authority to act not only on applications for original registration but also over all petitions filed
after original registration of title, with power to hear and determine all questions arising upon such
applications or petitions.
5

The principal action filed by INK in Civil Case No. Q-90-6937 before the trial court was for specific
performance with damages based on a document of sale. Such action was well within the exclusive
jurisdictions of the Regional Trial Court.  When IDP, the defendant in the trial court, did not question the
6
genuineness and validity of said deed of sale and its obligations thereunder, the summary judgment issued
by the court granting the reliefs sought by INK was also an exercise of its general jurisdiction.

Hence, when INK filed a motion for the issuance of an order from the same court to compel the holder of
the duplicate certificates of title to surrender the same to the Register of Deeds for the registration of the
deed of sale subject of the principal action, the motion was a necessary incident to the main case. When
the sale of the property was upheld by the court in its judgment and the defendant was directed to comply
with its terms and conditions, the right of INK to have the same registered with the Register of Deeds could
not be disregarded. To assert and enjoy its right, INK should be allowed to seek the aid of the court to direct
the surrender of the certificates of title. Since Regional Trial Courts are courts of general jurisdiction, they
may therefore take cognizance of this case pursuant to such jurisdiction. 
 Even while Sec. 107 of P.D. 1529 speaks of a petition which can be filed by one who wants to compel
another to surrender the certificates of title to the Register of Deeds, this does not preclude a party to a
pending case to include as incident therein the relief stated under Sec. 107, especially if the subject
certificates of title to be surrendered are intimately connected with the subject matter of the principal
action.  This principle is based on expediency and in accordance with the policy against multiplicity of suits.
8

The records of the case show that the subsisting mortgage lien of petitioner appears in the certificates of
title Nos. 26520 and 26521. Hence, the order of the trial court directing the surrender of the certificates to
the Register of Deeds in order that the deed of sale in favor of INK can be registered, cannot in any way
prejudice her rights and interests as a mortgagee of the lots. Any lien annotated on the previous certificates
of title which subsists should be incorporated in or carried over to the new transfer certificates of title. This
is true even in the case of a real estate mortgage because pursuant to Art. 2126 of the Civil Code it directly
and immediately subjects the property upon which it is imposed, whoever the possessor may be, to the
fulfillment of the obligation for whose security it was constituted. It is inseparable from the property
mortgaged as it is a right in rem — a lien on the property whoever its owner may be. It subsists
notwithstanding a change in ownership; in short, the personality of the owner is disregarded. Thus, all
subsequent purchasers must respect the mortgage whether the transfer to them be with or without the
consent of the mortgagee, for such mortgage until discharged follows the property.  It is clear therefore that
9

the surrender by petitioner of the certificates of title to the Register of Deeds as ordered by the trial court
will not create any substantial injustice to her. To grant the petition and compel INK to file a new action in
order to obtain the same reliefs it asked in the motion before the trial court is to encourage litigations where
no substantial rights are prejudiced. This end should be avoided. Courts should not be so strict about
procedural lapses that do not really impair the proper administration of justice. The rules are intended to
insure the orderly conduct of litigations because of the higher objective they seek, which is, to protect the
parties' substantive rights.  10

WHEREFORE, the appealed decision of the Court of Appeals dated 28 October 1992 is AFFIRMED.

INTESTATE ESTATE OF THE LATE DON MARIANO SAN PEDRO Y ESTEBAN, represented by its HEIR-JUDICIAL ADMINISTRATOR,
ENGRACIO F. SAN PEDRO, petitioner-appellant,
vs.
COURT OF APPEALS (Second Division) AURELIO OCAMPO, DOMINADOR D. BUHAIN, TERESA C. DELA CRUZ, respondents-appellees.
G.R. No. 106496 December 18, 1996
ENGRACIO SAN PEDRO, CANDIDO GENER, ROSA PANTALEON, VICENTE PANTALEON, ELEUTERIO PANTALEON, TRINIDAD SAN PEDRO,
RODRIGO SAN PEDRO, RICARDO NICOLAS, FELISA NICOLAS, and LEONA SAN PEDRO, petitioners,
vs.
THE HONORABLE COURT OF APPEALS, (Sixteenth Division) and REPUBLIC OF THE PHILIPPINES, respondents.

FACTS:

The case involves two petitions which were consolidated by the court in its decision.
1. GR 103727

Engracio San Pedro, as heir-judicial administrator of Plaintiff Intestate, filed a complaint for recovery of real property/ reconveyance
with damages and prayer for preliminary injunction against private defendants Ocampo, Buhain and dela Cruz.

San Pedro alleged that defendants acquired portion of the subject estate by employing fraud, bad faith and misrepresentation.

RTC of QC dismissed the complaint saying that the defendants are already the registered owners covered by the Torrens Title - which
cannot be defeated by the alleged Spanish Title of San Pedro. The Spanish Title also stated that the estate shall be excluded from the
coverage of Titulo Propriedad No. 4136. The court ordered Plaintiff Intestate to pay each defendant the amount of 5,000 and atty fees.

Motion for Recon was denied.  Petitioner filed an appeal,  CA dismissed.  

2. GR 106496

Engracio San Pedro and Justino Benito filed a petition for letter of administration over the intestate to be appointed as administrator
and co-administrator. Judge Echeverri appointed San Pedro as administrator and the court issued letter of administration in his favor
upon posting a bond of 10,000.

Republic of the Philippines filed a motion for intervention and opposition to the petition, claiming that the Titulo de Propriedad is
inadmissible and ineffective proof of ownership in court and it is invalid.

Republic filed a motion to suspend the proceedings but the Republic‘s opposition to the petition for letter of administration was
dismissed. Republic filed Motion for Recon.

The Judge declared Titulo de Propriedad as null and void and excluded all lands covered from the inventory of the estate of the late
Mariano San Pedro.

Petitioner-heirs appealed to CA. CA dismissed.

ISSUES:

1.Whether or not the lower court committed grave abuse of discretion amounting to lack of jurisdiction in settling the issue of
ownership of the estate covered by Titulo de Propriedad No. 4136?

2. Whether or not the lower court committed error in excluding from the inventory of the estate all lands covered by Titulo de
Propriedad No. 4136 on the ground that it is null and void?

RATIO:

1. NO. It is within the jurisdiction of the lower court functioning as probate court. The jurisdiction of the Probate Court is not limited to
the determination of who the heirs are and what shares are due them. Their main function is to settle and liquidate the estate of the
deceased so as to rule on whether the inventory of the estate properly included them for distribution of the net assets estate to lawful
heirs.

2. NO. The lower court did not commit any error when it declared Titulo de Propriedad No. 4136 as null and void, consequently
excluding all lands covered by the said title from the inventory of the estate.

Under PD 892, the system of registration under Spanish Mortgage Law was abolished and all holders of Spanish Titles should cause their
lands to be registered under Land Registration Act within 6 months from date of effectivity or until August 16, 1976.

In both cases, petitioner-heirs did not adduce evidence to show that Titulo de Propriedad No. 4136 was brought under the operation of
PD 892. There was no certificate of title shown.

Also, Titulo de Propriedad No. 4136, under PD 892, is inadmissible and ineffective as evidence of private ownership in special
proceedings case. Since the Titulo was not registered under Land Registration Act, said Titulo is inferior to the registered title of
defendants Ocampo, Buhain and dela Cruz. Torrens title of the latter enjoys the conclusive presumption of validity.
Petitioner-heirs failed to present neither the original Titulo nor a genuine copy thereof (only an alleged illegible copy was presented).
Even the secondary evidence presented was also not admissible.

RULING:

The Titulo de Propriedad is null and void and no rights can be derived therefrom. All lands covered by said Titulo are excluded from
inventory of the estate. The petition for letter of administration closed and terminated. The heirs are disallowed to exercise any act of
possession or ownership and ordered to vacate.

GUARANTEED HOMES, INC.  vs. HEIRS OF MARIA P. VALDEZ, et.al. 


G.R. No. 171531            January 30, 2009
Facts: 
Respondents, who are the descendants of Pablo Pascua (Pablo), filed a complaint seeking reconveyance of a
parcel of land situated in Subic, Zambales and covered by Original Certificate of Title (OCT) No. 404 in the name
of Pablo. In the alternative, the respondents prayed that damages be awarded in their favor.
OCT No. 404  was attached as one of the annexes of respondents’ complaint. It contained several annotations in
the memorandum of encumbrances which showed that the property had already been sold by Pablo during
his lifetime to Alejandria Marquinez and Restituto Morales. Respondents also attached copies of the following
documents as integral parts of their complaint: Transfer Certificate of Titles, the Extrajudicial Settlement of a
Sole Heir and Confirmation of Sales executed by Cipriano Pascua, Sr. (Cipriano), and the Deed of Sale with
Mortgage between spouses Albino Rodolfo and Fabia Rodolfo (spouses Rodolfo) and Guaranteed Homes, Inc..
It was further averred in the complaint that Jorge Pascua, Sr., son of Cipriano, filed on 24 January 1997 a
petition before the RTC of Olongapo City, Branch 75, for the issuance of a new owner’s duplicate of OCT No.
404. The RTC denied the petition. 
The trial court held that petitioner was already the owner of the land, noting that the failure to annotate the
subsequent transfer of the property to it at the back of OCT No. 404 did not affect its title to the property.
Petitioner filed a motion to dismiss the complaint on the grounds that the action is barred by the Statute of
Limitations, more than 28 years having elapsed from the issuance of TCT No. T-10863 up to the filing of the
complaint, and that the complaint states no cause of action as it is an innocent purchaser for value, it having
relied on the clean title of the spouses Rodolfo.
The RTC granted petitioner’s motion to dismiss. Noting that respondents had never claimed nor established
that they have been in possession of the property and that they did not present any evidence to show that
petitioner has not been in possession of the property either, the RTC applied the doctrine that an action to
quiet title prescribes where the plaintiff is not in possession of the property.
The Court of Appeals reversed the RTC’s order. In ordering the reinstatement of the complaint, the appellate
court ruled that the averments in respondents’ complaint before the RTC make out a case for quieting of title
which has not prescribed. Respondents did not have to prove possession over the property since petitioner as
the movant in a motion to dismiss hypothetically admitted the truth of the allegations in the complaint.
Issue: Whether or not petitioner is an innocent purchaser for value that there is no need to go beyond the
registered title of spouses Rodolfo.
Ruling: 
 It is basic that a person dealing with registered property need not go beyond, but only has to rely on, the title
of his predecessor-in-interest.

Since "the act of registration is the operative act to convey or affect the land insofar as third persons are
concerned," it follows that where there is nothing in the certificate of title to indicate any cloud or vice in the
ownership of the property, or any encumbrance thereon, the purchaser is not required to explore farther than
what the Torrens title upon its face indicates in quest for any hidden defect or inchoate right that may
subsequently defeat his right thereto.

If the rule were otherwise, the efficacy and conclusiveness of the certificate of title which the Torrens system
seeks to insure would entirely be futile and nugatory. The public shall then be denied of its foremost motivation
for respecting and observing the Torrens system of registration. In the end, the business community stands to
be inconvenienced and prejudiced immeasurably.
In this case, it is enough that petitioner had examined the latest certificate of title which in this case was issued
in the name of the immediate transferor, the spouses Rodolfo. The purchaser is not bound by the original
certificate but only by the certificate of title of the person from whom he had purchased the property.
Then, while the Extrajudicial Settlement of a Sole Heir and Confirmation of Sales executed by Cipriano alone
despite the existence of the other heirs of Pablo, is not binding on such other heirs, nevertheless, it has
operative effect under Section 44 of the Property Registration Decree.
Even assuming arguendo that the extrajudicial settlement was a forgery, the Court still has to uphold the title of
petitioner. The case law is that although generally a forged or fraudulent deed is a nullity and conveys no title,
there are instances when such a fraudulent document may become the root of a valid title.
Registration in the public registry is notice to the whole world. Every conveyance, mortgage, lease, lien,
attachment, order, judgment, instrument or entry affecting registered land shall be, if registered, filed or
entered in the Office of the Register of Deeds of the province or city where the land to which it relates lies, be
constructive notice to all persons from the time of such registering, filing or entering
Note: Not entirely related to the issue, but in case he ask.
Respondents cannot make out a case for quieting of title since OCT No. 404 had already been cancelled.
Respondents have no title to anchor their complaint on.Title to real property refers to that upon which
ownership is based. It is the evidence of the right of the owner or the extent of his interest, by which means he
can maintain control and, as a rule, assert right to exclusive possession and enjoyment of the property.
Respondents’ claim against the Assurance Fund also cannot prosper. Section 101 of P.D. No. 1529 clearly
provides that the Assurance Fund shall not be liable for any loss, damage or deprivation of any right or interest
in land which may have been caused by a breach of trust, whether express, implied or constructive. Even
assuming arguendo that they are entitled to claim against the Assurance Fund, the respondents’ claim has
already prescribed since any action for compensation against the Assurance Fund must be brought within a
period of six (6) years from the time the right to bring such action first occurred, which in this case was in 1967.

BARANDA VS GUSTILO
GR 81163, SEPTEMBER 26, 1988

FACTS:

A petition for reconstitution of title was filed with the CFI (now RTC) of Iloilo involving a parcel of land known as
Lot No. 4517 of the Sta. Barbara Cadastre covered by OCT No. 6406 in the name of Romana Hitalia.
The OCT was cancelled and TCT No. 106098 was issued in the names of petitioners Baranda and Hitalia.

The Court issued a writ of possession which Gregorio Perez, Maria P. Gotera and Susana Silao refused to honor on
the ground that they also have TCT No. 25772 over the same Lot No. 4517.

The Court found out that TCT No. 257772 was fraudulently acquired by Perez, Gotera and Susana.

Thereafter, the court issued a writ of demolition which was questioned by Perez and others so a motion for
reconsideration was filed.

Another case was filed by Baranda and Hitalia (GR. NO. 62042) for the execution of judgement in the resolutions
issued by the courts.

In the meantime, the CA dismissed a civil case (GR. NO. 00827) involving the same properties. (NOTE: This
time three cases na ang involve excluding the case at bar.)

The petitioners prayed that an order be released to cancel No.T-25772. Likewise to cancel No.T-106098
and once cancelled to issue new certificates of title to each of Eduardo S. Baranda and Alfonso Hitalia To
cancel No.T-25772. Likewise to cancel No.T-106098 and once cancelled to issue new certificates of title to
each of Eduardo S. Baranda and Alfonso Hitalia.
In compliance with the order or the RTC, the Acting Register of Deeds Avito Saclauso annotated the order declaring
TCT T-25772 null and void, cancelled the same and issued new certificate of titles in the name of petitioners. 

However, by reason of a separate case pending in the Court of Appeals, a notice of lis pendens was annotated in the new
certificate of title. 

This prompted the petitioners to move for the cancellation of the notice of lis pendens in the new certificates. 

Judge Tito Gustilo then ordered the Acting Register of Deeds for the cancellation of the notice of lis pendens but the
Acting Register of Deeds filed a motion for reconsideration invoking Sec 77 of PD 1529.

ISSUE: What is the nature of the duty of a Register of Deeds to annotate or annul a notice of lis pendens in a torrens
certificate of title.

HELD: 

Section 10, Presidential Decree No. 1529 states that "It shall be the duty of the Register of Deeds to immediately
register an instrument presented for registration dealing with real or personal property which complies with all the
requisites for registration. ... If the instrument is not registrable, he shall forthwith deny registration thereof and
inform the presentor of such denial in writing, stating the ground or reasons therefore, and advising him of his right
to appeal by consulta in accordance with Section 117 of this Decree."

Section 117 provides that "When the Register of Deeds is in doubt with regard to the proper step to be taken or
memoranda to be made in pursuance of any deed, mortgage or other instrument presented to him for registration
or where any party in interest does not agree with the action taken by the Register of Deeds with reference to any
such instrument, the question shall be submitted to the Commission of Land Registration by the Register of Deeds, or
by the party in interest thru the Register of Deeds. ... ."

The function of ROD is ministerial in nature


The function of a Register of Deeds with reference to the registration of deeds encumbrances, instruments and the
like is ministerial in nature. The respondent Acting Register of Deeds did not have any legal standing to file a motion
for reconsideration of the respondent Judge's Order directing him to cancel the notice of lis pendens annotated in
the certificates of titles of the petitioners over the subject parcel of land. 

In case of doubt as to the proper step to be taken in pursuance of any deed ... or other instrument presented to him,
he should have asked the opinion of the Commissioner of Land Registration now, the Administrator of the National
Land Title and Deeds Registration Administration in accordance with Section 117 of Presidential Decree No. 1529.
No room for construction for the laws on functions of ROD
The elementary rule in statutory construction is that when the words and phrases of the statute are clear and
unequivocal, their meaning must be determined from the language employed and the statute must be taken to
mean exactly what it says. The statute concerning the function of the Register of Deeds to register instruments in a
torrens certificate of title is clear and leaves no room for construction

De leon v Deleon

FACTS:
a. Bonifacio O. De Leon, while still single , purchased from the [PHHC] through a Conditional Contract to Sell on July 20, 1965 a
parcel of land with an area of 191.30 square meters situated in Fairview, Quezon City for P841.72;
 
b. On April 24, 1968, Bonifacio O. De Leon married plaintiff Anita B. De Leon before the Municipal Mayor of Zaragosa, Nueva
Ecija. Both parties stipulate that said marriage is valid and binding under the laws of the Philippines;
 
c. On June 22, 1970, Bonifacio O. De Leon paid [PHHC] the total amount of P1,023.74 x x x. The right of ownership over the
subject parcel of land was transferred to the late Bonifacio O. De Leon on June 22, 1970, upon the full payment of the total
[price] of P1,023.74 and upon execution of the Final Deed of Sale;
 
d. After full payment, Bonifacio O. De Leon was issued [TCT] No. 173677 on February 24, 1972;
 
e. On January 12, 1974, Bonifacio O. De Leon executed a Deed of Sale in favor of defendants-spouses Felix Rio Tarrosa and Lita
O. De Leon disposing the parcel of land under TCT No. 173677 for valuable consideration amount of P19,000.00 and
subscribed before Atty. Salvador R. Aguinaldo who was commissioned to [notarize] documents on said date. The parties
stipulate that the Deed of Sale is valid and genuine. However, plaintiff Anita De Leon was not a signatory to the Deed of Sale
executed on January 12, 1974;
 
f. That plaintiff Anita B. De Leon and the late Bonifacio O. De Leon were married in church rites on May 23, 1977 x x x;
 
g. The late Bonifacio O. De Leon died on February 29, 1996 at the UST Hospital, España, Manila;
 
h. The said "Deed of Sale" executed on January 12, 1974 was registered on May 8, 1996 before the Office of the Register of
Deeds of Quezon City and [TCT] No. N-173911 was issued to Lita O. De Leon and Felix Rio Tarrosa. 5

ISSUE: 

1.

WON THE SUBJECT PROPERTY IS CONJUGAL

2.
3.

Whether 1/2 of the conjugal assets do not vest to Bonifacio because of the absence of liquidation

4.

RULING: 
1.

 The full payment of the conditional contract was during marriage, thus ownership was transferred only during the
marriage. 

2.

NCC 160, the governing provision at the time of Bonifacio and Anita’s marriage, provides that properties acquired during marriage
are presumed to belong to the conjugal partnership unless it is proved that it pertains exclusively to the husband or the wife. It is not
even necessary to prove that the property was acquired with the funds of the partnership, for only proof of acquisition during the
marriage is needed to raise the presumption that the property is conjugal. Even when the manner in which the properties were acquired
does not appear, the presumption will apply.

Here, ownership over what was once a PHHC lot and covered by the PHHC-Bonifacio Conditional Contract to Sell was only
transferred during the marriage of Bonifacio and Anita. Conditional sale is akin, if not equivalent to a contract to sell - ownership is
retained by the seller and is not passed to the buyer until full payment of the price, unlike in a contract of sale where title passes upon
delivery of the thing sold. The efficacy or obligatory force of the vendor's obligation to transfer title is conditioned upon full payment; if
the condition has not been fulfilled, the conditional obligation would stand as it had never existed.

Evidently, title to the property in question only passed to Bonifacio after he had fully paid the purchase price on June 22, 1970. 
This full payment was made more than two (2) years after his marriage to Anita on April 24, 1968.   In net effect, the property was
acquired during the existence of the marriage; as such, ownership to the property is, by law, presumed to belong to the conjugal
partnership

It is not exclusive just because it was registered solely in his name. The mere registration of a property in the name of one spouse
does not destroy its conjugal nature. What is material is the time when the property was acquired. Thus, they were not able to
overthrow the presumption of the conjugal nature of the property as no evidence was brought forth to prove that the source of funding
solely came from Bonifacio.

The deed of sale is also void ab initio for not having marital consent from Anita as provided by NCC 166 . Since Art. 166 of the
Code requires the consent of the wife before the husband may alienate or encumber any real property of the conjugal partnership, it
follows that the acts or transactions executed against this mandatory provision are void except when the law itself authorized their
validity.

1.

Sale of one-half of the conjugal property without liquidation of the partnership is void.

2.

Prior to liquidation, right of the husband or wife in the conjugal assets is inchoate (mere expectancy) and does not ripen into a
title until it appears that there are assets in the community as a result of the liquidation and settlement . Their interest is limited to the
net remainder (remanente liquido) resulting from the liquidation after dissolution. Thus, their right will only be determined by the net
assets left after settlement of obligations which can be divided by the spouses or their heirs.

YOLANDA O. ALFONSO v. OFFICE OF THE PRESIDENT and PHIL-VILLE


DEVELOPMENT AND HOUSING CORPORATION
520 SCRA 64 (2007), SECOND DIVISION
Petitioner Yolanda O. Alfonso (Alfonso), then the register of deeds of Caloocan City, was found
administratively liable for allegedly acquiescing to the change of the date of
the registration of OCT No. 994 from May 3, 1917 to April 19, 1917, and for making it
appear that there were two OCT Nos. 994. Consequently, she was dismissed
from government service for grave misconduct and dishonesty.

Alfonso was investigated by the Land Registration Authority (LRA) upon the request of Phil-
Ville Development Corporation (Phil-Ville) who purchased some parts of the land. Phil-Ville‘s
letter-complaint led to the conduct of an inquiry by the Senate Committees on Justice
and Human Rights, and on Urban Planning, Housing and Resettlement which found that
Alfonso acted maliciously, fraudulently and in bad faith hence it recommended the filing
of administrative cases against her and her conspirators.

On the other hand, LRA finds her guilty of Grave Misconduct and recommended her
dismissal.

Upon review by the Department of Justice (DOJ) of the LRA decision, it recommended to
the Office of the President (OP) that Alfonso, a presidential appointee, be found guilty of
Grave Misconduct and Dishonesty and be dismissed from the service.

Consequently, OP issued an Administrative Order ordering the dismissal of Alfonso.

Court of Appeals (CA) affirmed the Administrative Order. Alfonso came to Supreme Court to


seek a reversal of the CA‘s Decision and its Resolution affirming her dismissal ordered by OP.

ISSUE:
Whether or not the Court of Appeals erred in upholding decision of the Office of the President
because Alfonso‘s right to due process was violated

HELD:
Alfonso was given every opportunity to explain her side and to present evidence in her defense
during the administrative investigation conducted by the LRA. Records sufficiently show that
in compliance with the “show-cause” letter of the LRA Administrator, she submitted her
written explanation, and that during the pre-trial conferences, she presented
documentary evidence.
Likewise, the quantum of proof required in an administrative proceeding is only
substantial evidence or that amount of relevant evidence that a reasonable mind might accept
as adequate to support a conclusion. The standard of substantial evidence is satisfied when
there is reasonable ground to believe that the person indicted was responsible for the alleged
wrongdoing or misconduct. In the case at bar, petitioner stood charged not for changing the
date of registration of OCT No. 994 but rather, she was indicted for acquiescing to the change
by (1) issuing conflicting “certifications” on the date of issuance of OCT No. 994; and (2) for
making it appear that there were two OCT Nos. 994. Thus, her protestations that she had no
hand in the alteration are unavailing.
 

NAAWAN COMMUNITY RURAL BANK INC., Petitioner, 


vs. 
THE COURT OF APPEALS and SPOUSES ALFREDO AND ANNABELLE LUMO, Respondents.

PRINCIPLES:
A person dealing with registered land may generally rely on the correctness of a certificate of title and
the law will in no way oblige him to go beyond it to determine the legal status of the property.
Registration of instruments must be done in the proper registry in order to bind the land.
Lumos filed an action for quieting of title. 

DECISION:
The Lumo are adjudged the absolute owners and possessors of the properties in question, and all
improvements thereon and quieting title thereto as against any and all adverse claims of the
defendant. Further, the sheriffs certificate of sale, Sheriffs deed of final conveyance, Tax Declarations,
and any and all instrument, record, claim, encumbrance or proceeding in favor of the defendant, as
against the plaintiffs, and their predecessor-in-interest, which may be extant in the office of the
Register of Deeds of Province of Misamis Oriental, and of Cagayan de Oro City, and in the City
Assessors Office of Cagayan de Oro City, are declared as invalid and ineffective as against the
plaintiffs title.

FACTS:
On April 30, 1988, a certain Guillermo Comayas offered to sell to spouses Alfredo and Annabelle
Lumo, a house and lot located in Cagayan de Oro City.
Wanting to buy the property, Lumos made inquiries at the Office of the Register of Deeds and the
Bureau of Lands in Cagayan on the legal status of the vendors title. They found out that the property
was mortgaged to a certain Mrs. Galupo and that the owners copy of the Certificate of Title to said
property was in her possession.
Lumos directed Comayas to redeem the property. As a result, the property was releasde of the
adverse claim of Galupo as annotated on TCT.
On May 17, 1988, the Lumos and Guillermo Comayas, executed a deed of absolute sale which later
on was registered and inscribed on TCT No. T-41499 and, on even date, TCT No. T-50134 was issued
in favor of the Lumos.
After obtaining their TCT, the Lumos requested the issuance of a new tax declaration certificate in
their names. However, they learn from the City Assessors Office that the property was also declared
for tax purposes in the name of  Naawan Community Rural Bank Inc.
It was found out that on February 7, 1983, Guillermo Comayas obtained loan from petitioner Bank
using the subject property as security. At the time said contract of mortgage was entered into, the
subject property was then an unregistered parcel of residential land recorded in the registration
book of the Province of Misamis Oriental, not in the registration book of Cagayan de Oro City. 
It appears that, when the registration was made, there was only one Register of Deeds for the entire
province of Misamis Oriental, including Cagayan de Oro City. It was only in 1985 when the Office of
the Register of Deeds for Cagayan de Oro City was established separately from the Office of the
Register of Deeds for the Province of Misamis Oriental.
Failure to pay, the real estate mortgage was foreclosed and the subject property sold at a public
auction to the mortgagee Naawan Community Rural Bank as the highest bidder. Thereafter, the
sheriffs certificate of sale was issued and registered under Act 3344 in the Register of Deeds of
the Province of Misamis Oriental.
On April 17, 1984, the subject property was registered in original proceedings under the Land
Registration Act. Title was entered in the registration book of the Register of Deeds of Cagayan de
Oro City as Original Certificate of Title, pursuant to Decree No. N-189413.(It is now under the Torrens
System)
On July 23, 1984, Transfer Certificate of Title No. T-41499 in the name of Guillermo P. Comayas was
entered in the Register of Deeds of Cagayan de Oro City.
Meanwhile, on September 5, 1986, the period for redemption of the foreclosed subject property lapsed
and the MTCC Deputy Sheriff of Cagayan de Oro City issued and delivered to petitioner bank
the sheriffs deed of final conveyance. This time, the deed was registered under Act 3344 and
recorded in the registration book of the Register of Deeds of Cagayan de Oro City. 
On January 27, 1989, the Regional Trial Court issued an order for the issuance of a writ of execution
of its judgment which promptly issued by MTCC.
However, when the writ was served, the property was no longer occupied by Comayas but herein
private respondents, the spouses Lumo who had, as earlier mentioned, bought it from Comayas on
May 17, 1988
Alarmed by the prospect of being ejected from their home, Lumos filed an action for quieting of
title. 
RTC rendered a decision declaring private respondents as purchasers for value and in good faith,
and consequently declaring them as the absolute owners and possessors of the subject house and lot.
CA affirmed.
Petitioner bank contends that the earlier registration of the sheriffs deed of final conveyance in the day
book under Act 3344 should prevail over the later registration of private respondents deed of absolute
sale under Act 496 as amended by the Property Registration Decree, PD 1529.

ISSUES: 
I. WON the sheriffs deed of final conveyance was duly executed and registered in the Register of
Deeds of Cagayan de Oro City on Dec. 2 1986. 
II. WON registration of sheriff’s deed of final conveyance in the ROD could be effective as against
spouses Lumo.
III. Whether or not private respondents could be considered as buyers in good faith.

RULING:
Article 1544 of the Civil Code governs the double sale of immovable property which provides that,
should it be immovable property, the ownership shall belong to the person acquiring it who in good faith first
recorded it in the Registry of Property, was invoked by the parties.
.
I. No, it was not duly registered. 
It has been held that, where a person claims to have superior proprietary rights over another on the
ground that he derived his title from a sheriffs sale registered in the Registry of Property, Article 1544
of the Civil Code will apply only if said execution sale of real estate is registered under Act 496.
In Bautista vs. Fule where the Court ruled that the registration of an instrument involving unregistered
land in the Registry of Deeds creates constructive notice and binds third person who may
subsequently deal with the same property.
At the time of the execution and delivery of the sheriffs deed of final conveyance on September
5, 1986, the disputed property was already covered by the Land Registration Act and Original
Certificate of Title pursuant to Decree No. N189413 was likewise already entered in the registration
book of the Register of Deeds of Cagayan De Oro City as of April 17, 1984.
Thus, the inscription of the unregistered land under Act No, 3344 was held not effective. 
II. No, registration of instrument affecting titled lands under Act No. 3344 is ineffective against
3 Person.
rd

When Lumos bought the subject property, the same was already registered under the Torrens System.
Under the said system, registration is the operative act that gives validity to the transfer or creates a
lien upon the land. The issuance of a certificate of title had the effect of relieving the land of all
claims except those noted thereon. It is a well-known rule in this jurisdiction that persons dealing
with registered land have the legal right to rely on the face of the Torrens Certificate of Title
and to dispense with the need to inquire further, except when the party concerned has actual
knowledge of facts and circumstances that would impel a reasonably cautious man to make
such inquiry.
Private respondents were not required by law to go beyond the register to determine the legal
condition of the property. They were only charged with notice of such burdens on the property as were
noted on the register or the certificate of title. To have required them to do more would have been
to defeat the primary object of the Torrens System which is to make the Torrens Title
indefeasible and valid against the whole world.

III. Whether or not private respondents could be considered as buyers in good faith.
Allegation:
The due and proper registration of the sheriffs deed of final conveyance on December
2, 1986 amounted to constructive notice to private respondents. Thus, when private
respondents bought the subject property on May 17, 1988, they were deemed to have
purchased the said property with the knowledge that it was already registered in the name of
petitioner bank.
Answer:
Yes, Lumos exercise the required diligence in ascertaining the legal condition of the title
to the subject property so as to be considered as innocent purchasers for value and in good
faith.
Before private respondents bought the subject property, inquiries were made with the
Registry of Deeds and the Bureau of Lands regarding the status of the vendors title. No liens or
encumbrances were found to have been annotated on the certificate of title. Neither were
private respondents aware of any adverse claim or lien on the property other than the adverse
claim of a certain Geneva Galupo to whom Guillermo Comayas had mortgaged the subject
property which eventually settled and the adverse claim previously annotated on the title
cancelled. Thus, having made the necessary inquiries, private respondents did not have to
go beyond the certificate of title. Otherwise, the efficacy and conclusiveness of the
Torrens Certificate of Title would be rendered futile and nugatory.
Considering therefore that private respondents exercised the diligence required by law
in ascertaining the legal status of the Torrens title of Guillermo Comayas over the subject
property and found no flaws therein, they should be considered as innocent purchasers for
value and in good faith.

OVERALL CONCLUSION:
The priority in time principle being invoked by petitioner bank is misplaced because its registration
referred to land not within the Torrens System but under Act 3344.

You might also like